Domanda:
Perché la fusione fredda è considerata fasulla?
Sklivvz
2011-01-25 03:33:19 UTC
view on stackexchange narkive permalink

La fusione fredda è stata citata molto ultimamente a causa di alcune nuove impostazioni che apparentemente funzionano. Questa è un'affermazione non verificata.

Vedi ad esempio:

Mentre dovremmo dare alla comunità scientifica il tempo di valutare l'impostazione e alla fine replicare i risultati, c'è indubbiamente un certo scetticismo sul fatto che la fusione fredda avrebbe funzionato, perché l'affermazione è piuttosto straordinaria.

In passato, dopo che Fleischmann e Pons hanno annunciato i loro risultati di fusione fredda , in perfetta buona fede, sono stati smentiti da esperimenti successivi.

W Quali sono le realtà sperimentali che rendono facili gli esperimenti sulle fusioni fredde in stile Fleischmann e Pons?

Gli stessi rischi si applicherebbero a questo nuovo allestimento?

Il tuo link * Journal of Nuclear Physics * mi restituisce 403 e non riesco a trovarne un altro sul sito web. Il sito web sembra che il Journal esista allo scopo di riferire sulla fusione fredda.
A parte le questioni tecniche, quella "carta" di Rossi e Focardi dovrebbe far scattare nella tua testa tutti i tipi di campanelli d'allarme: la scrittura amatoriale, la mancanza di descrizione della configurazione sperimentale, la mancanza di dati, l'enfasi sull'invenzione e sui brevetti, il fatto che il "giornale di fisica nucleare" non sembra essere altro che il blog di qualcuno ...
@dmckee,@nibot Anche per me ora è 403-ing. So che la "rivista" non è una pubblicazione autorevole. Ammettono di averlo impostato perché non potevano essere pubblicati (hanno accusato lo stigma della FC). In ogni caso il documento non includeva il setup sperimentale (dicono solo di aver usato un calorimetro, fondamentalmente), ma il setup è mostrato nella domanda di brevetto. Sì, anche a me sembra fasullo.
"apparentemente funziona" "perfettamente in buona fede" Sei troppo soggettivo. La ragione per cui la maggior parte dei fisici è molto scettica è perché teoricamente non ha senso.
Un diario gestito da solo non è la fine della strada, ma invita a un attento esame. Sapere di essere sotto il microscopio e svolgere un lavoro meno che impeccabile non ispira fiducia.
@Jeff: In teoria non ha senso e le prove sperimentali sono scarse nella migliore delle ipotesi.
Perché il voto chiuso? L'ho eseguito attraverso meta e mi sembra che la domanda stia producendo risposte oggettive?
@Sklivvz: Penso che sia anche una bella domanda appartenente al sito. Saluti
Il fatto che la fusione fredda sia un mucchio di **** fumante non è una giustificazione per chiudere la discussione sul perché è un mucchio di **** fumante.
Inoltre, tieni presente che i brevetti per le _ideas_ originali non funzionano. Anche l'ufficio brevetti degli Stati Uniti ha rilasciato migliaia di brevetti per le macchine a moto perpetuo. Un brevetto non significa nulla in questo contesto.
Vedi: [Fusione fredda: la tecnologia più controversa della scienza è tornata] (https://www.newscientist.com/article/mg23130910-300-cold-fusion-sciences-most-controversial-technology-is-back/)
Tredici risposte:
Ron Maimon
2011-08-19 01:01:30 UTC
view on stackexchange narkive permalink

Il trattamento riservato a Pons / Fleischmann è di gran lunga il peggior scandalo della scienza moderna. Probabilmente è peggio di Galileo. Le loro affermazioni erano vere, sono state riprodotte immediatamente in diversi laboratori (sporadicamente, anche molti laboratori hanno fallito) e la buona ricerca continua ancora oggi, senza finanziamenti e senza teoria.

Storia sperimentale

La loro affermazione non è completamente nuova. Risale agli anni '20, quando Paneth e Peters segnalarono la produzione di elio nell'elettrolisi dell'acqua pesante Pd. Le affermazioni di Paneth e Peters furono respinte essenzialmente per la stessa ragione per cui lo furono quelle di Pons e Fleischmann, l'immaginazione dei teorici era troppo stentata per escogitare un meccanismo che potesse colmare il divario tra energie chimiche ed energie nucleari. Negli anni '50, c'era uno scienziato sovietico che affermava anche che l'elettrolisi dell'acqua pesante Pd porta ad anomalie nucleari. Questa affermazione è stata respinta anche dall'establishment scientifico sovietico, ma la sua carriera è stata riabilitata un po 'dopo che gli americani hanno riprodotto l'effetto nel 1989.

Molte persone (cioè studenti laureati) che hanno lavorato con il sistema Pd / deuterio hanno notato anomalie nel sistema per decenni, ed era folclore nella comunità chimica che il palladio deuterato agisse, mentre il Pd idrogenato no. Pons e Fleischmann decisero di prendere sul serio le anomalie, e per molti anni fecero una calorimetria estremamente attenta sul sistema, finché furono certi che avevano un effetto riproducibile per il quale la chimica poteva essere esclusa senza problemi. Poi hanno tenuto la conferenza stampa e il caos.

Parte del problema è che una volta dichiarata la fusione, la gente ha insistito sul fatto che la fusione avrebbe dovuto emettere neutroni, proprio come fa la fusione calda. Questo è impossibile, perché, considerando l'energia rilasciata, il numero di neutroni avrebbe cucinato Pons e Fleischmann. Quindi i fisici nucleari chiesero di misurare gli effetti nucleari e tentarono di farlo, ma le loro misurazioni nucleari erano piene di errori ed è possibile che abbiano falsificato un complotto mostrato a una conferenza (sebbene considerando l'impeccabile integrità scientifica di Fleischmann, trovo più plausibile che abbiano commesso un errore onesto). È importante notare che il loro articolo pubblicato contiene solo dati calorimetrici e nessun dato nucleare di cui non erano sicuri.

Alcune persone hanno ipotizzato che l'effetto possa essere spiegato dalla chimica, o da un'insufficiente agitazione, o da immagazzinare energia elettrica per un successivo rilascio. Queste affermazioni sono tutte idiote. L'effetto non è piccolo, l'unica ragione per cui richiede strumenti per rilevare è perché Pons e Fleischmann hanno deliberatamente usato un minuscolo filo di Pd come catodo. Quando hanno usato una piastra Pd più grande, la cosa ha fuso il tavolo e ha fatto un buco nel pavimento di cemento sottostante. Non esiste una fonte di energia chimica, né una batteria, che possa immagazzinare energia chimicamente a più di circa 1eV per atomo. Anche altre persone hanno notato esplosioni simili.

A parte l'esplosione, c'è un possibile calore dovuto alla ricombinazione, che è stata spesso enfatizzata dalla critica. Gli elettrodi separano H2 da O2 e se i due si mescolano e l'idrogeno brucia, vedrai calore in eccesso. Per controllare ciò, i gruppi hanno utilizzato telecamere a infrarossi per individuare la fonte di calore al catodo, piuttosto che l'acqua in cui le bolle di gas possono mescolarsi. Hanno anche separato l'anodo e il catodo. Ma in modo più definitivo, nel 1994, Pons e Fleischman hanno dimostrato il calore dopo la morte nel 1994, dove aumentano la temperatura nella cella in modo anomalo, quindi interrompono completamente la corrente. La cella continua a produrre calore per ore senza nessuna corrente , senza ossigeno, senza idrogeno e molte volte più calore di quanto si possa immagazzinare nel catodo con qualsiasi mezzo chimico.

l'effetto è molto sensibile alla metallurgia del palladio, e Pons e Fleischmann non sono riusciti a riprodurre l'effetto schizzinoso su richiesta una volta esaurito il palladio buono. L'esperimento a volte richiede settimane e molte persone semplicemente non hanno avuto pazienza. Tuttavia, l'effetto è stato riprodotto immediatamente in una manciata di posti. Il MIT ha eseguito una famigerata riproduzione che ha notato la produzione di calore in eccesso e stava per stampare con una riproduzione. Poi si sono resi conto che questo effetto sarebbe stato etichettato come fasullo e hanno tagliato il grafico per non mostrare il calore in eccesso. Uno degli studenti laureati coinvolti in questo esperimento, Eugene Mallove, era così indignato che lasciò la sua posizione e divenne un promotore della fusione fredda.

Diversi gruppi hanno pubblicato riproduzioni. Questi gruppi sono stati attaccati nel modo più antiscientifico. Diversi gruppi, Bocris in Texas, ma anche rinomati ricercatori dell'istituto Bhabha e Los Alamos, hanno riportato bassi livelli di produzione di trizio nel sistema. Poiché il trizio è radioattivo, ha una firma chiara, non può essere scambiato per nient'altro. Poiché è molto costoso ed è normalmente prodotto nei reattori nucleari, l'unico modo in cui un segnale del genere potrebbe essere visto è se fosse stato deliberatamente simulato aggiungendo trizio all'acqua pesante. Bocris è stato accusato di aver fatto proprio questo --- arricchire le sue cellule con il trizio, così fiducioso erano i negatori che aveva commesso una frode. Nonostante l'intensa pressione, non ha mai ritirato la sua richiesta. Un altro collega in Texas che ha affermato che il trizio, Wolf, ha ritirato l'affermazione quando ha visto cosa stava succedendo a Bocris, e non ha mai più parlato a sostegno della fusione fredda. Bocris è stato indagato per cattiva condotta scientifica ed esonerato. Non è mai stato trovato alcun modo plausibile per ottenere il trizio (a parte la fusione fredda). Le osservazioni del trizio richiederebbero tutti i ricercatori che hanno osservato il trizio di essere coinvolti in una frode deliberata. È impossibile identificare erroneamente il trizio.

Due teorici estremamente rispettati, Julian Schwinger (emerito UCLA) e Peter Hagelstein del MIT, erano convinti che l'effetto fosse reale. Schwinger non è stato autorizzato a pubblicare sul campo e Hagelstein, che era di ruolo, ha avuto tutti i suoi finanziamenti tagliati ed è stato spostato in un armadio.

All'inizio degli anni '90, senza alcun finanziamento ufficiale, McKubre ha quantificato la produzione di elio per correlarla al calore in eccesso. Si osservano bassi livelli di fusione ordinaria nel sistema. Il gruppo SPAWAR della Marina degli Stati Uniti ha riprodotto l'effetto in esperimenti di co-deposizione, dove placca Pd su una superficie in presenza di acqua pesante. I loro esperimenti rilevano principalmente prodotti nucleari, perché la superficie placcata è molto piccola, ma gli effetti sono riproducibili al 100%. Più recentemente, la marina ha presentato prove di sporadici neutroni ad alta energia provenienti dal sistema di co-deposizione.

In Giappone, Mizuno ha notato che nuovi elementi venivano prodotti nel sistema Pd con rapporti isotopici anormali e numero atomico vicino a Pd (questo è stato rilevato anche da Wolf, come riportato da Eugene Mallove, ma Wolf non avrebbe pubblicato dopo il fiasco del trizio). Rapporti isotopici anomali non possono essere frodi, perché tali materiali sono così difficili da realizzare. In Giappone, Arata ha riprodotto l'effetto utilizzando il caricamento di gas di deuterio in Pd, che non ha fonte di calore, quindi non c'è alcun errore calorimetrico a cui dare la colpa. Questa era una versione infallibile degli esperimenti di "calore dopo la morte" di Pons e Fleischmann, ed esclude anche del tutto l'errore di calorimetria / ricombinazione. L'effetto è stato riprodotto molte centinaia di volte, in laboratori lontani senza interessi reciproci, e tutti dovrebbero ormai essere certi che sia reale. Mi vergogno di me stesso, in quanto non sono riuscito a fidarmi dei dati sperimentali fino a quando non ho escogitato una ragionevole storia teorica per spiegarlo.

Le affermazioni di Focardi / Rossi sono più dubbie. Il loro effetto è sul nichel / idrogeno, che ha anche riportato alcune anomalie energetiche, ma non con lo stesso livello di sicurezza. In termini di follia teorica, la fusione del nichel idrogeno sta alla fusione del palladio deuterio come la fusione del palladio deuterio sta alla fusione calda standard. La comunità della fusione fredda sta assumendo un atteggiamento attendista, ma penso che il consenso sia che è improbabile che il dispositivo funzioni. Nelle sue dimostrazioni, Rossi ha misurato la produzione di calore usando il vapore, non l'acqua, e sottovalutando il contenuto di acqua del vapore, è possibile gonfiare l'energia prodotta dall'energia latente della vaporizzazione, che è enorme. Per me, è molto sospetto che i suoi prodotti di trasmutazione dichiarati siano stati analizzati e abbiano rapporti isotopici naturali. È possibile che la sua macchina funzioni, ed è possibile che non produca affatto energia in eccesso, lo sapremo abbastanza presto. Ciò che è impossibile è che non ci siano effetti nucleari in Pd / deuterio.

Ecco un aggiornamento riguardante l'e-cat, che, come le persone si aspettavano, è una truffa sofisticata: È l'E- cat di Andrea Rossi et al. sul serio?.

Lavoro teorico

Una delle maggiori difficoltà per l'accettazione dell'effetto è che il lavoro teorico in questo campo non è valido. Esistono diverse teorie, ognuna delle quali è più o meno assurda. Le difficoltà centrali sono superare in qualche modo la barriera di Coulomb e produrre energia senza sottoprodotti della reazione nucleare:

  • Idrini / poco idrogeno: questa teoria afferma che l'elettrone nell'idrogeno può trovare un'orbita più vicina dello stato fondamentale e trascorre alcune volte vicino al nucleo. Ciò richiede che la meccanica quantistica sia sbagliata, o che ci sia una nuova forza di elettroni / protoni che è stata persa, e in qualche modo non altera l'energia dello stato fondamentale, ma è in grado di risucchiare l'elettrone nel protone ogni tanto.
  • Deuteroni / alfa condensati di Bose-Einstein: questa idea è che la sezione d'urto per la fusione sia potenziata da effetti di particelle identiche, poiché i deuteroni e gli alfa sono entrambi bosoni. In teoria, puoi potenziare le reazioni facendo in modo che una sorgente coerente di bosoni attraversi la stessa reazione a una sovrapposizione coerente. Questa teoria fallisce sia perché la temperatura è troppo alta per la coerenza tra i deuteroni, sia perché quando è implementata in specifiche carte di fusione fredda, i deuteroni vengono trattati come particelle non interagenti in uno stato di prodotto, in modo che l'ampiezza sia la stessa il punto è grande. Ma questo sta ignorando l'intera difficoltà, perché la repulsione elettrostatica porta la funzione d'onda ad essere impigliata, con poche probabilità che due deuteroni qualsiasi arrivino allo stesso punto.
  • Meccanismi di potenziamento del reticolo: questo era l'obiettivo di Schwinger e Hagelstein, nessuno dei quali ha affermato di aver risolto il problema. Il problema con tali teorie è solo che gli effetti devono essere collettivi su migliaia di atomi per spiegare che si prendono energie eV in energie KeV, ed è termodinamicamente difficile immaginare come si possa portare tale energia entropica in un posto così sfavorevole entropicamente come un singolo particella.
  • Produzione di neutroni a forza debole: la teoria di Widom Larson afferma che è possibile per un protone e un elettrone eseguire il decadimento beta inverso sulla superficie di un metallo, dove sono presenti grandi campi elettrici locali. Questo è assurdo, a causa della differenza MeV nella massa di protoni e neutroni. Sono necessari milioni di volt per accelerare un elettrone a un'energia sufficiente per essere in grado di eseguire un decadimento beta inverso e tali energie non sono disponibili sulla superficie di un metallo. Inoltre, questa teoria predice le trasmutazioni di più / meno un'unità di massa in modo predominante, che non viene osservata, e non spiega come un deuterone possa assorbire un elettrone.

Le seguenti liste sono teorie fasulle che ho ipotizzato avrebbero funzionato, anche altre persone escogitano queste ogni tanto:

  • Cattura sporadica di muoni atmosferici: l'idea è che i muoni vengano catturati dal metallo e conducono alla fusione. Questo non funziona, solo perché non ci sono abbastanza muoni, i deuteroni sono separati l'uno dall'altro nel reticolo e se il muone viene catturato da un nucleo di Pd, viene sprecato.
  • Tunneling con molti strani -Miglioramento del corpo: l'idea è che l'ampiezza del tunneling sia sempre stimata, non calcolata, e questo è un sistema con molti elettroni / molti nucleoni impossibile da risolvere, quindi forse l'ampiezza del tunnel è appena fuori di molti ordini di grandezza. Questo non funziona, perché c'è un modo per dare un limite inferiore alle ampiezze di tunneling che esclude qualsiasi reazione di fusione apprezzabile dal tunneling. Per fare questo, si sfrutta il fatto che il tunneling è una proprietà dello stato fondamentale, e i deuteroni che si immaginano tunnel sono bosoni e il loro stato fondamentale del tempo immaginario non ha nodi. Gli elettroni hanno nodi, poiché sono fermioni, e ad alte energie, ma quando gli stati elettronici sono tutti completamente occupati, potrebbero anche essere un vuoto, con struttura solo vicino alla superficie di Fermi (questo segue dall'approssimativo simmetrico particella-buco descrizione del liquido Fermi). C'erano limiti superiori rigorosi sulla probabilità di tunneling per deuteroni in un metallo che sosteneva di dimostrare che la fusione fredda è impossibile.

I precedenti fallimenti inducono ad aspettarsi che l'effetto sia fuori equilibrio, e coinvolge atomi molto eccitati.

La mia teoria personale

Per colmare il divario tra la scala della chimica a eVs e dei nuclei a MeVs, si dovrebbe prendere atto del fatto che ci sono elettroni K-shell che orbitano molto vicino al nucleo a energie KeV. L'elettrone K-shell di Pd ha un'energia di ionizzazione di 20KeV, e se hai un buco K-shell in un atomo di Pd, immagazzina una quantità di energia in modo non entropico in una quantità sufficiente per portare alla fusione del deuterone. Sebbene questa energia sia grande, non è abbastanza grande da far cadere un atomo di palladio fuori dalla sua posizione reticolare, quindi non può scaricare la sua energia rompendo localmente il reticolo. Il motivo è che il nucleo Pd richiede più dell'energia di ionizzazione 20KeV per essere messo fuori posizione senza il suo nucleo, e non puoi trasferire in modo cospiratorio l'energia del foro all'intero nucleo in un unico passaggio, è impossibile lo spazio delle fasi.

Tali buchi del guscio K di solito decadono dai raggi X, ma questo è un processo elettromagnetico che viene soppresso dalle potenze di v / c quando l'elettrone non è relativistico, come lo è anche nel guscio K. Questo è un effetto ben noto: è la stessa ragione per cui le linee spettrali atomiche sono strette. L'emissione di un fotone richiede molte orbite a causa del disadattamento di scala tra la lunghezza d'onda del fotone e la dimensione dell'orbita. Questo in definitiva è perché l'orbita non è relativistica. Poiché l'emissione richiede così tanto tempo, le righe spettrali sono nettamente definite e strette e l'emissione è dominata dagli elementi della matrice del momento di dipolo dello stato atomico tra stati stazionari.

Altri modi osservati per K- Per i gusci perdere la loro energia significa espellere un elettrone guscio esterno da un atomo vicino. Questo processo è elettrostatico e non relativistico, quindi non è soppresso da fattori 1 / c. Viene soppresso solo dalla piccolezza della carica sull'elettrone e dalla distanza tra gli elettroni sugli atomi vicini. C'è una frazione significativa di decadimenti nei buchi K in Pd in ​​questo canale.

In un metallo con protoni o deuteroni, un foro del guscio K dovrebbe essere in grado di trasferire la sua energia in un protone o deutroni anche tramite forze elettrostatiche. L'elemento della matrice è esattamente lo stesso che per calciare un elettrone, ma la densità degli stati è 30-50 volte più grande (a seconda che sia un protone o un deuterone) a causa della massa più pesante. Il protone, a differenza di un nucleo Pd, lascerà il suo sito reticolare sotto tale trasferimento. Quindi, considerando che la sezione trasversale di un buco del guscio K per calciare un elettrone non è piccola, mi sento sicuro di concludere che il processo di espulsione del protone è il meccanismo di decadimento dominante per i buchi K.

Questi i deuteroni hanno esattamente la stessa energia del buco del guscio K, il che significa che il loro punto di svolta classico quando ci si avvicina a un nucleo di Pd è esattamente la stessa distanza dal nucleo elettrostaticamente come il guscio K è largo, circa 100 fermis. Questi buchi possono quindi eccitare un altro elettrone in modo coerente e viaggiare per molti passaggi nel reticolo prima di decadere dai raggi X allo stato fondamentale. Questi stati buco-deuterone creano bande di diverse larghezze KeV a energie intorno a 20 KeV, e queste bande sono piene di punti di svolta classici a 100fermis da un nucleo Pd.

Ora supponiamo che due di questi deuteroni accelerati accadano vicino allo stesso nucleo di Pd. Questo può facilmente produrre un evento di fusione al punto di svolta, dopotutto i deuteroni hanno circa 20 KeV e le velocità di fusione a 20 KeV in fasci non sono così piccole, figuriamoci nei casi in cui la funzione d'onda è concentrata vicino a un nucleo con un punto di svolta classico (dove la funzione d'onda è potenziata).

Questa fusione non avviene necessariamente nel solito modo di fusione calda, poiché è molto vicina a un nucleo di Pd. Supponiamo che la fusione trasferisca elettrostaticamente l'energia / quantità di moto in eccesso a una particella carica vicina, il candidato ovvio è uno dei nuclei protonici Pd. Quindi la particella alfa e qualunque cosa a cui ha trasferito la sua energia si muovono insieme con 24MeV di energia e attraversano il metallo, ionizzando gli atomi di Pd. Dal punto di vista energetico, possono realizzare fino a 1000 fori K-shell, tutti entro un millimetro, poiché la profondità di penetrazione è così piccola. Il numero vero è più probabilmente un centinaio o qualche centinaio, poiché tutti i livelli sono eccitati durante il processo Bethe di ionizzazione delle particelle cariche. Questi fori vengono quindi fasciati con deuteroni, quindi accelerano nuovi deuteroni e questo può facilmente portare a una reazione a catena. Credo che questo spieghi la fusione fredda.

Ci sono due problemi con questa idea:

  1. La sezione trasversale per la fusione a 20 KeV non è così enorme, e lo fa non porta da solo a una reazione a catena attraverso i soliti canali di fusione calda. Il fattore di moltiplicazione è di circa 0,001 dalla fusione del fascio su Pd deuterato, che ha una percentuale di successo di 1 su 100.000, non 1 su 100, a 20 KeV.
  2. La reazione effettiva osservata produce una particella alfa senza un neutrone emesso o protone quasi tutto il tempo. Questo è un evento su un milione nella fusione calda.

Penso che entrambi i problemi siano legati al fatto che la reazione avviene all'interno di un metallo denso. Il primo problema non è presente se due deuteroni sono fasciati ed entrambi girano intorno ad un nucleo, il risultato è come una collisione diretta di due fasci da 20KeV con un ottimo dispositivo di focalizzazione (il nucleo) per concentrare la funzione d'onda di scattering.

La fusione dei deuteroni avviene sempre attraverso stati intermedi instabili e la sezione d'urto della particella alfa è piccola solo a causa dello stesso problema non relativistico. Per ottenere un alfa, è necessario emettere un fotone di raggi gamma e le emissioni di fotoni vengono soppresse da fattori 1 / c. Quando c'è un nucleo vicino, può essere espulso elettrostaticamente, e questo processo è più facile che espellere un fotone, perché non è relativistico (lo stesso vale per un elettrone, ma con una sezione trasversale molto più piccola a causa della carica più piccola, e lì non c'è motivo di sospettare la concentrazione della funzione d'onda attorno alla densità elettronica, come nel caso di un nucleo).

La scala temporale per calciare un nucleo è la durata della risonanza dei due deuteroni, che non è molto lunga , in termini di distanza, è di circa 100 fermi, circa della stessa dimensione del guscio interno. Se i deuteroni si muovono a caso, questa coincidenza non è significativa, ma se le eccitazioni del deuterone-foro sono fasciate, è plausibile che quasi tutte le collisioni energetiche deuterone-deuterone avvengano molto vicino a un nucleo, come spiegato sopra. / p>

Ci sono leggi di conservazione infrante quando un nucleo è vicino. Il nucleo rompe la parità, quindi potrebbe aprire un canale di fusione, consentendo alle coppie di deuteroni di decadere in un alfa da uno stato dispari di parità. Una tale transizione non sarebbe mai stata osservata in una fusione a fascio diluito, perché queste fusioni avvengono lontano da qualsiasi altra cosa. Questa ipotesi non è esclusa dalla spettroscopia di particelle alfa (ci sono molti livelli rilevanti di parità diverse), ma non è nemmeno prevista.

Ma poiché qualcosa deve spiegare il dati sperimentali, e questa idea è l'unica storia che non è completamente inverosimile, credo che questo sia ciò che sta succedendo.

Questa teoria prevede quanto segue

  • durante la fusione fredda, il materiale dovrebbe emettere abbondanti raggi X nell'intervallo KeV (da quei fori del guscio K che si decadono comunque elettromagneticamente).
  • il materiale dovrebbe emettere deuteroni KeV in un mm di pelle intorno ad esso.
  • il materiale dovrebbe emettere alfa di gamma MeV e frammenti nucleari Pd, protoni, detonori, trizio e prodotti di fusione Pd, come appropriato per lo scattering di elettroni ~ 10MeV. Gli alfa dovrebbero arrivare fino a 20MeV, che è l'energia massima quando l'intero nucleo è disperso. I frammenti Pd dovrebbero essere energia MeV.
  • Ci dovrebbe essere una piccola quantità di fusione calda che si verifica, con i neutroni veloci e il trizio associati, solo dalle occasionali collisioni accidentali di fusione calda di deuteroni 20KeV lontani da un nucleo. Se le bande diventano incoerenti, puoi ottenere un'esplosione di neutroni, poiché i deuteroni veloci incoerenti si fondono in modo casuale.

Queste erano "previsioni" solo in quanto non le conoscevo quando le ho fatte la teoria. Ho scoperto su lenr-canr.org che 1,3,4 sono osservati da Mosier-Boss usando rivelatori di plastica CR-39 e pellicola a raggi X, insieme ad altri, anche se è difficile vedere un deuterone KeV.

La teoria prevede anche quanto segue

  • La fusione fredda basata su protoni non funziona (sebbene potrebbe esserci un modo per immagazzinare le energie della scala KeV in un sistema di idrogeno di nichel per lungo tempo in Bande K-shell, rilasciandolo a raffiche, anche se mi sembra improbabile). Ciò richiede che tutti i rapporti sul calore in eccesso della fusione fredda Ni-H siano dovuti alla ricombinazione chimica, nessuno di essi dovrebbe mostrare alcun prodotto nucleare. Questo non è incoerente con i dati che ho visto.
  • I prodotti di trasmutazione nella fusione fredda sono dovuti alla frammentazione del Pd durante la fusione e all'assorbimento / diffusione veloce dell'alfa o all'assorbimento / diffusione del frammento di Pd veloce.

Le previsioni di frammentazione e assorbimento alfa più semplici significano che dovresti osservare la transizione del Pd ad Au (+1, dall'assorbimento alfa e dall'eiezione del protone, o l'assorbimento alfa, l'emissione gamma e il decadimento beta) e +2 Cd (dall'emissione gamma dell'assorbimento alfa ), ma non superiore e le transizioni verso il basso sono dovute alla frammentazione, quindi dovresti vedere i prodotti di fissione Pd, Rh (dai protoni espulsi) e Ru (alfa espulsi). Questi sono precisamente coerenti con i dati di trasmutazione di Wolf, che provenivano dallo spettro gamma dei radioisotopi presenti nel catodo dopo un ciclo di successo, come trapelato da Eugene Mallove.

Ma queste previsioni non sono compatibili con tutte le sperimentazioni dati presentati a lenr-canr.org riguardanti le trasmutazioni. Credo che, nella misura in cui i dati di trasmutazione sono in disaccordo con questa teoria, è sbagliato.

Trasmutazioni

I prodotti di trasmutazione in deuterio forzati attraverso Pd presentati da mostrano picchi a massa +8, +12. Il bombardamento da parte di alfa non può produrlo, poiché non c'è assolutamente alcuna possibilità che lo stesso atomo venga colpito due volte da due diversi alfa.

Ciò richiede che un Pd frammentato da fusione espelle un Be8 poiché è un prodotto di frammentazione della fusione, e che questo Be8 è stato poi assorbito da un altro nucleo in transito, dando 8 unità di massa ad un altro nucleo per assorbimento. Se è così, ci sono regole di somma per gli elementi di trasmutazione: la quantità di elemento di luce X prodotto è uguale alla differenza di (Pd + X) e (Pd-X), dove Pd + X significa sommare tutti i protoni e neutroni in X a Pd, e Pd-X significa sottrarre tutti i neutroni e protoni in X da Pd. Questa regola della somma è un test rigoroso della teoria.

Inoltre, se si assume che la probabilità di assorbimento sia approssimativamente geometrica, deve interrompersi esattamente quando la barriera di Coulomb supera 10MeV, il che significa trasmutazioni di (questo è ragionevole per l'assorbimento di un piccolo proiettile nucleare da parte di un grande nucleo), è possibile concludi che rileverai picchi a Pd + X sistematicamente spostati dagli stessi fattori di Pd-X.

Ma ci si aspetterebbe che i prodotti di trasmutazione pesante cadano come soppressione della barriera coulombiana. Questo è solo qualitativamente plausibile date le osservazioni di Iwamura. Puoi vedere i risultati delle specifiche di massa di Iwamura su lenr-canr.org

Questa teoria è più o meno una teoria di massa, quindi è difficile capire perché la superficie è più importante. Si prevede che le bande a 20 KeV si verifichino nei metalli deuterati, e questo darebbe un intero zoo di effetti utili, indipendentemente dalla sostanza nucleare. La larghezza di banda sposta lo spettro dei raggi X del metallo deuterato da 20KeV a un'ampia gamma, che è anche una predizione facilmente verificata della teoria: le frequenze di risonanza del guscio K sono alterate dalla deutrazione.

Sto raccontando questa storia teorica perché penso che sia plausibile e coerente con i dati senza una nuova fisica fondamentale, in modo che le persone non dovrebbero respingere la fusione fredda. Poiché nessun'altra spiegazione si avvicina neanche lontanamente al lavoro, sospetto che questa spiegazione sia corretta.

Problemi con la teoria

Il problema principale con la teoria è l'incompletezza, è uno schizzo. Ma i punti principali dei dati sperimentali non sono più in conflitto con la teoria. Questa è una modifica importante al post originale, che ho inserito qualche tempo fa. Al momento del post originale, non mi è venuto in mente che trasferire elettrostaticamente 10 MeV di energia a un nucleo può portare alla frammentazione nucleare, e senza questo, la teoria è incompatibile con i dati di trasmutazione.

Hm, questo in realtà suona abbastanza ragionevole. D'ora in poi terrò una mente un po 'più aperta riguardo alla fusione fredda. Grazie per la risposta informativa.
Sembri un po 'prevenuto per la fusione fredda, così come io sono prevenuto contro di essa. Ho lavorato per molti degli elettrochimici incapaci di riprodurre il lavoro di Pons / Fleischmann. Questo non significa che P / F fosse sbagliato, ma sarei molto sorpreso se questi eccellenti scienziati con cui ho lavorato si fossero persi qualcosa. Forse hai investito denaro / tempo nella fusione fredda se ci credi davvero, ma sto coprendo le mie scommesse investendo altrove. Non importa quanto sia buona la tua idea, ma importa cosa funziona davvero. Finora non vedo un vincitore dalla tua parte, ma possa vincere l'idea migliore!
"Penso che entrambi i problemi siano legati al fatto che la reazione sta avvenendo all'interno di un metallo denso". Penso che entrambi i problemi siano legati al fatto che la reazione non sta avvenendo affatto.
@Chris: Sono di parte solo per la fusione fredda perché mi sento in colpa per non averci creduto prima. Non ho alcun interesse personale, a parte il fatto che credo che la spiegazione che ho dato sopra sia quella corretta.
@Ben: È importante leggere i documenti sperimentali prima di prendere una decisione e rivederli con occhio critico (lenr-canr.org).
Se è un effetto reale, perché non investire qualche migliaio di dollari in modo da poter utilizzare l'effetto per far bollire l'acqua, guidare una turbina e diventare milionario? Perché nessuno lo fa? Se hai trovato la fusione fredda, costruisci il dannato reattore , o almeno mostra di aver prodotto del litio o dell'elio o qualcosa del genere. Non ci sarebbero dubbi se qualcuno costruisse il dannato reattore .
L'effetto è difficile da riprodurre. Pons e Fleischmann erano sicuri di poter costruire un reattore, ma avevano abbastanza difficoltà a ottenere l'effetto una volta alla settimana. Se avessi dei soldi, mi piacerebbe creare un sistema Pd / d e provare a guidarlo con i raggi X sintonizzati sulle transizioni del guscio interno, per vedere se i gusci interni stanno facendo qualcosa di strano. Personalmente, sarei felice di avere abbastanza dati sperimentali per fare un'efficace teoria del campo delle risonanze del deuterone in un forte campo elettrico. O sarebbe fisica interessante in sé e per sé.
@RonMaimon: Non ti piace Widom-Larsen, ma non ti piacciono nemmeno le implicazioni della tua spiegazione preferita, in parte a causa della vasta gamma di isotopi che le persone vedono. Capisco che ci siano molte ricerche che hanno portato a una migliore comprensione della metallicità stellare derivante dalla nucleosintesi del processo r, nonché i risultati dell'analisi di attivazione dei neutroni. Quale potenziale vede nell'applicare la ricerca in questi campi alla spettroscopia degli esperimenti LENR al fine di eliminare alcuni dei gradi di libertà che tormentano le spiegazioni esistenti?
@Eric: L'ampia gamma di isotopi è estremamente sconcertante, ma non posso escluderla definitivamente. Potrebbe plausibilmente essere dovuto al bombardamento di 20Mev alfa / Pd che distrugge i nuclei. Puoi testare questa idea direttamente facendo esperimenti di bombardamento. Widom Larson è un no-go: non puoi creare neutroni, è solo energeticamente impossibile. Trovo tutte le spiegazioni esistenti allo stesso modo ridicole, motivo per cui ho sentito di dover fornire un'alternativa ragionevole. Non conosco il processo r, ma da Wiki, sembra richiedere un ambiente ricco di neutroni. Come si tradurrebbe? Tutte le idee sono benvenute.
Stavo pensando di presumere che gli spettri riportati fossero accurati entro un errore ragionevole, e quindi di lavorare a ritroso da lì per eliminare potenziali catene di decadimento. Eventuali passaggi intermedi renderebbero questa una sfida difficile, ma, dal punto di vista ingenuo di un principiante, non una sfida impossibile. Se si fosse fortunati, si potrebbe potenzialmente escludere un gran numero di catene, lasciando un sottoinsieme che potrebbe essere utilizzato per fissare un po 'di più le cose. Non sarebbe necessario fare troppe supposizioni sul processo iniziale - bombardamento alfa, ecc. L'attenzione si concentrerebbe principalmente sul decadimento beta.
@Eric: Sono d'accordo che puoi eliminare alcune catene. Non sono sicuro che ci sia una contabilità completa dello spettro di reazione Pd / d che è quantitativa, se lo trovi, pubblicheresti un link? Quella che ho trovato era una spettrometria più qualitativa sulla diffusione forzata del deuterio attraverso Pd, se ricordo bene.
@Ron: Di seguito è riportato un collegamento a un documento con il tipo di dati che pensavo sarebbe stato interessante analizzare: http://bit.ly/rTA47W. Stanno combinando ampi dati SIMS con NAA ad alta risoluzione. lenr-canr ha una serie di link a documenti che menzionano SIMS e NAA, e una rapida occhiata indica che ci sono almeno alcuni grafici abbastanza ampi che sono disponibili. Devo ancora trovare un set di dati adeguato che possa essere analizzato senza troppi problemi, ma spero che siano disponibili da qualche parte. Questo documento suggerisce che i nuclei di Pd-108 si fissano in elementi più leggeri: http://bit.ly/vm6z4e.
Ho apportato _maggiori_ modifiche qui, poiché ho capito qualcosa di ovvio che mi mancava prima.
@EricWalker: Se sei ancora qui, capisco _ completamente_ le trasmutazioni. Posso dirti quali esperimenti sono buoni e quali sono cattivi, uno per uno.
Nel classico esperimento di tipo F & P, da dove vengono questi buchi K-shell e perché apparentemente non compaiono (o non fanno nulla) la maggior parte del tempo? E perché allora i raggi X emessi vengono rilevati così raramente?
@KirkShanahan: I fori del guscio K devono essere seminati per avviare la reazione, sia dai raggi cosmici, sia da una radioattività naturale da qualche parte, quindi possono decadere dai raggi X o andando a una banda inferiore. Le bande superiori sono moltiplicativamente instabili per fusione, vengono reintegrate dalle particelle cariche MeV prodotte dalla fusione, ma hanno bisogno di un seme. Penso che questo sia il motivo principale per cui la reazione è sporadica, è necessaria una contaminazione radiativa per avviarla. I raggi X vengono emessi solo quando c'è un eccesso di calore e non ce ne sono così tanti, solo molto per fusione. Il film li rileva regolarmente.
@KirkShanahan: Andare a una banda inferiore con un elettrone che assorbe l'energia, piuttosto che un raggio X. Questo è un compromesso, non è chiaro quale effetto domini, ma si vedono i raggi X emessi ben al di sopra dello sfondo in co-deposizione (la pellicola di nebbia catodica), e lo spettro generale è KeV-ish (Mosier-Boss), questo è stato uno dei principali indizi su cosa sta succedendo.
@KirkShanahan: si ritiene che i raggi X siano collimati. Supponendo che ciò fosse vero, forse potrebbero essere persi se il rilevatore non fosse nella posizione giusta.
@RonMaimon: eventuali suggerimenti sugli esperimenti di trasmutazione sono i benvenuti.
@EricWalker: Le trasmutazioni sono frammenti nucleari dello spettatore Pd nella fusione, che vengono emessi e assorbiti da altri Pd. Questo è il motivo per cui vedi picchi a +/- 8 +/- 12 +/- 16 e picchi a ogni +/- numero pari. Emettere nuclei luminosi di questo tipo è facile, questi sono i frammenti più stabili (questo è spiegato nella nuova versione della risposta, leggi le modifiche). La cosa che pensavo stesse uccidendo la teoria - le grandi trasmutazioni - ne sono attualmente la migliore prova. Scriverò qualcosa di formale al riguardo.
@EricWalker: La collimazione è un po 'eccessiva, ma il modo più semplice per vedere è circondare il catodo con pellicola e cercare macchie. Non ho visto alcun rapporto di punti nelle foto ai raggi X. Sono sicuro che ci sono emissioni di raggi X a basso livello dell'ordine di 10 fotoni per fusione per tutto il tempo in cui il calore in eccesso è in corso. Non sono molti fotoni per joule, poiché c'è 24 MeV di calore per fusione e solo 240KeV in fotoni a raggi X per fusione, che è l'1% di potenza nei fotoni a raggi X, potrebbe essere fino al 10% del calore in eccesso (se tutta la banda passa ai raggi X), ma questo è il limite massimo estremo. Sono pochi deciwatt.
@RonMaimon: Prenderò la tua parola sulle energie coinvolte nell'emissione di raggi X. Per la collimazione, vedere Karabut, Karabut and Hagelstein, http://www.iscmns.org/CMNS/JCMNS-Vol6.pdf, p. 217 sgg.
Il problema con l'utilizzo di pellicole sensibili ai raggi X negli esperimenti di fusione fredda, in particolare nelle celle F&P, è che anche il calore provoca l'appannamento. Le celle vengono normalmente eseguite a> r.t. per tempi molto lunghi, e da quello che ricordo le pellicole sono tenute il più vicino possibile per catturare il maggior numero possibile di raggi X, il che massimizza l'esposizione termica. Inoltre, i raggi X collimati darebbero punti chiari sulla pellicola, mentre tutti i film che ho visto mostrano macchie amorfe ad eccezione di quelle del BHARC. Lì vedono modelli di spot reali, ma c'è una semplice spiegazione per questo: hanno usato accidentalmente una pellicola esposta.
@KirkShanahan: C'è un'altra possibilità che i raggi X vengano emessi nei punti di fusione e vengano diffratti dal cristallo, cosa che trovo molto più probabile. Negli esperimenti SPAWAR, il film è posizionato abbastanza lontano dalla regione di codeposizione da rendere impossibile l'esposizione al calore. Mi fido delle attente osservazioni della gente della fusione fredda sulla speculazione incurante di persone che hanno inventato ragioni per negare il fenomeno (in altre parole, non mi fido del tuo giudizio su questo, poiché hai sbagliato in modo monumentale in passato e devi ancora invertire te stesso).
@Ron - No, non ho ancora "sbagliato in modo monumentale" nulla relativo a questo. Tuttavia, hai prodotto un mucchio di informazioni errate e ricorri all'assassinio del personaggio (come hai appena fatto ancora una volta) invece di una discussione ragionata. Se pensi che non sia corretto, provalo. * In particolare * indica un errore e * spiega perché è * con più di un semplice gesto della mano. Solo uno ... tutto qui.
@KirkShanahan: Ok --- prendi le dozzine di osservazioni al trizio. Sono semplicemente impossibili senza frode, poiché non puoi produrre il trizio chimicamente e non puoi falsificare il suo rilevamento (è radioattivo con una firma chiara). Bocris, Wolf, Bhabha e Los Alamos hanno rilevato il trizio. Fine della storia. Fatto. Nient'altro necessario. Amico, mi stai davvero facendo incazzare. Ridendo delle prove, del trizio, e inventando spiegazioni a castello per il resto, rendi impossibile avere una discussione razionale. Non ci proverò più, ho cose da fare.
Amico, non sto ignorando nulla, mi rifiuto semplicemente di credere ciecamente in una nuova fisica quando * so * che la contaminazione, le interferenze e la cattiva tecnica analitica possono rovinare facilmente numeri come questi. Ho bisogno di più prove di un mucchio di numeri estremamente variabili. E ne vedo le prove! Voglio vedere il controllo. Se possono mai controllare l'effetto e prevedere quale T otterranno in quali condizioni, allora hanno fatto qualcosa. NON è dove siamo oggi e hanno avuto quasi 25 anni per non fare progressi. Ciò suggerisce che le loro premesse sono sbagliate. È ora di un nuovo albero.
@Ron: si prega di inviare un collegamento al tuo articolo quando una bozza è disponibile.
@EricWalker: Proverò a scrivere qualcosa durante il Ringraziamento --- Sto facendo del lavoro di bioinformatica, e questo richiede molto tempo, poiché richiede molta programmazione. Sto anche avendo difficoltà a trovare buoni dati di prodotto per 20MeV elettrone / fotone su Pd, anche se ho trovato i modelli generali di espulsione, non ho rapporti di frammentazione quantitativa. Questo è un po 'fastidioso, ci sono molti dati, è solo difficile cercarli. È difficile prevedere teoricamente i rapporti di frammentazione.
@RonMaimon: Due commenti interessanti sulla tua proposta: http://bit.ly/ThCPfj e http://bit.ly/SgBOqi.
@EricWalker: Spaandonk ha un'intuizione sbagliata: l'energia depositata da un alfa 24 MeV non viene consumata rapidamente dagli elettroni di valenza, è preferenzialmente depositata in K-shell (questo è descritto dalla formula di ionizzazione di Bethe). Il risultato non è "colla", particelle cariche di questa energia attraversano un mm di metallo prima di fermarsi e lasciano dietro di sé un mm di centinaia di atomi ionizzati K-shell. Questo K-shell è mescolato con D nel metallo e forma una band. Il K-shell da 20KeV non può eliminare un nucleo di Pd, perché un tale nucleo sarebbe completamente ionizzato, e questa è più energia della sola ionizzazione del K-shell.
... entrambe queste cose sono state le prime cose che ho controllato anni fa, e ho trovato il fattore di moltiplicazione richiesto dagli esperimenti di fusione diluita in travi, ed era a pochi ordini di grandezza (come 2 o 3, non 10). Questo non è stato scoraggiante, perché le altre teorie sono scostate di 10 ordini di grandezza, e anche la mancanza di neutroni significa che sta accadendo qualcos'altro. La reazione a doppia d richiede che la banda K-shell conduca i deuteroni agli urti sulla superficie, dove sono sufficientemente concentrati per consentire la fusione. Il Ni-H (se esiste) richiede che si verifichi la reazione d-p.
AGGIORNAMENTO (via e-mail): Ron vede ora che il limite di Hagelstein proibisce una reazione incoerente.Questo esclude il processo incoerente che ha proposto come spiegazione per gli esperimenti sul calore in eccesso, la radiazione osservata è troppo bassa.Ciò indica processi coerenti, e ora sta esplorando versioni coerenti dello stesso processo, per completare il modello di Hagelstein, che ritiene del tutto valido, e se vengono identificate le modalità appropriate, dovrebbe spiegare il fenomeno.
@Jerry Schirmer: Se questo modello è corretto, nonostante sia fisicamente interessante, potrebbe _non_ essere economicamente sostenibile.
@Joshua: va bene.Ma i Tokamak non sono economicamente sostenibili, ma possono e sono stati costruiti .
sigoldberg1
2011-01-25 10:53:27 UTC
view on stackexchange narkive permalink

Questo ha avuto una bellissima risposta teorica subito alla sessione APS del 1989 a New York, penso da Koonin. Teoricamente, per qualsiasi tipo di fusione è necessario superare la repulsione coulombiana dei nuclei rilevanti, dell'ordine di MeV per consentire ai nuclei di avvicinarsi abbastanza da far sì che le loro funzioni d'onda si sovrappongano e si fondano. A causa del fenomeno del tunneling meccanico quantistico, questo può essere ridotto da decine a centinaia di kev. Quindi temperature di >> 10 ^ 5 K, o muoni freddi (che superano gli elettroni di 200 volte) sono necessarie per ridurre la distanza internucleare (come nella fusione fredda catalizzata da muoni, un fenomeno reale), o qualche altro meccanismo speciale è richiesto per consentire questo approccio ravvicinato.

Tuttavia, affinché avvenga qualsiasi tipo di fusione catalizzata chimicamente, cioè tramite gli elettroni di valenza, l'energia di legame dei due atomi di H al catalizzatore dovrebbe essere così alta, che la particolare configurazione di gli elettroni di valenza a bassa energia, ecc. sarebbero necessariamente del tutto irrilevanti per il problema, cioè qualunque sia la loro disposizione non potrebbero catalizzare i nuclei fusibili per avvicinarsi abbastanza da fondersi. Quindi nessuna disposizione intelligente di impacchettamento, quasiparticelle, adsorbimento speciale, strutture reticolari cristalline speciali, ecc. Potrebbero mai alterare questa conclusione. Qualunque cosa stesse accadendo a scale così basse di energia sarebbe apparsa come una specie di lanugine irrilevante rispetto alla scala energetica della distanza internucleare necessaria per la fusione.

Pertanto la fusione fredda catalizzata da elettroni di valenza violerebbe le leggi fondamentali della meccanica quantistica, della fisica nucleare, ecc. Anche Leggett e Baym pubblicarono un argomento come questo nello stesso periodo (riassunto gratuitamente qui). Koonin e Nauenberg hanno pubblicato un calcolo accurato qui, mostrando che se la massa dell'elettrone fosse 5-10 volte più grande di quanto non sia in realtà, la fusione chimicamente calalizzata potrebbe funzionare. Si noti tuttavia che la velocità di reazione dipende dalla massa dell'elettrone in modo molto, molto forte, in modo che ciò rimanga impossibile nel nostro universo.

10 kev funziona a causa del tunneling.
C'è una certa confusione tra la barriera di Coulomb e l'energia di legame qui?
@Anna, la barriera coulomb è l'energia minima che i nuclei devono superare per "fondersi". Questo è in termini di reazione chimica l'energia di attivazione.
Julian Schwinger ha cercato di presentare documenti sulla fusione fredda, ma sono stati respinti dagli arbitri. Avrebbero potuto fare letture interessanti poiché Julian era probabilmente lo studente più brillante di Robert Oppenheimer.
Dato che questo è apparso di nuovo nella prima pagina, aggiungerò il commento che faccio anche nella mia risposta: i cristalli possono comportarsi in modo diverso rispetto a quanto ci dice l'intuizione termodinamica e quantistica in blocco. Potrebbe esserci un modo in cui un "elettrone virtuale" potrebbe propagarsi lungo il reticolo cristallino che ha la massa maggiore necessaria che hai menzionato. Il problema con la fusione fredda non è che sia teoricamente impossibile, ma che non è stato dimostrato in modo coerente negli esperimenti. Altrimenti ci sarebbero macchine in vendita, come ci sono i pannelli solari. Quando la macchina esiste, arriva la teoria.
-1 questa risposta è miope. I dati sperimentali devono essere valutati mettendo da parte i pregiudizi teorici. Ci sono validi argomenti teorici per i deuteroni KeV nella fusione fredda, ma anche in mancanza di questa prova, l'argomento "l'alta energia richiede un'alta temperatura" è semplicemente falso.
"Quindi nessuna disposizione intelligente di impacchettamento, quasiparticelle, adsorbimento speciale, strutture reticolari cristalline speciali, ecc. Potrebbe mai alterare questa conclusione". Perfetto, ma tutto si basa sul fatto che il concetto di elettroni di valenza è un'approssimazione valida in un ampio insieme di composti chimici in circostanze adiabatiche. Se qualcosa nel sistema multielettrone rendesse la descrizione non valida, lo sarebbe anche questa conclusione.
Ricordo che alcuni anni fa i Quasicrystals erano impossibili da Theory e che, l'ormai premio Nobel 2011, Shechtman subiva molti pregiudizi a causa di idee preconcette di "impossibilità" (lo riferì in una video intervista). Le teorie sono provvisorie e devono essere sostituite da altre più perfette. Aspetterò e spero che sia vero per il bene di tutti noi. Di solito le teorie sono provvisorie e seguono gli esperimenti e non il contrario.
@annav: Non credo che la mancanza di dimostrazioni coerenti negli esperimenti dica molto sulla realtà di un presunto fenomeno sperimentale. Alcune cose sono difficili da definire. Ci sono voluti anni per capire come realizzare transistor affidabili, ad esempio.
@sigoldberg1: Stai affrontando la questione della fusione fredda da un punto di vista teorico - in pratica dicendo che la teoria non lo consente. Penso che sia molto positivo sottolinearlo. Ma in fondo, le affermazioni centrali relative alla fusione fredda sono attualmente empiriche. Per questo motivo, mi sembra che le critiche debbano affrontare i presunti fenomeni sperimentali e che argomentare dalla teoria stia facendo retrocedere le cose; per essere d'accordo con te, dobbiamo presumere che l'attuale teoria sia ferrea in questo settore.
@EricWalker Sono d'accordo, ed è per questo che tengo una mente aperta per essere convinto una volta che gli esperimenti diventano convincenti.
Non è mai stato catalizzato chimicamente.L'indagine è partita da un errore davvero semplice che coinvolgeva un campo elettrico troppo semplificato prodotto da cilindri metallici.(L'equazione semplice non è valida all'interno dei cilindri ma è stata applicata come se lo fosse.)
dmckee --- ex-moderator kitten
2011-01-25 04:44:19 UTC
view on stackexchange narkive permalink

Il dispositivo di Fleischmann e Pons si basava sulla calorimetria (misurazione del bilancio energetico in termini di calore) mantenuta su più intervalli di tempo per accertare che qualcosa di inaspettato stesse accadendo nella cella.

Questo è sperimentalmente complicato, poiché richiede misurazioni della temperatura ad alta precisione per essere mantenute rispetto a un riferimento coerente e si basa sulla comprensione di come il calorimetro possa o meno perdere calore attraverso canali non monitorati.

In breve, una grande perdita di calore viene sottratta da un grande apporto di calore ed entrambe le misurazioni hanno qualche incertezza. Questo è un segnale di avvertimento in qualsiasi esperimento, ma non una campana a morto se l'incertezza può essere quantificata con sufficiente precisione.

La situazione è complicata dalla perdita di calore dipendente dal tempo che Fleischmann e Pons ha riferito. Se reale, ciò indicherebbe un processo inaspettato al lavoro, anche se non abbiamo modo di sapere a priori se si tratta di fusione o di qualche meccanismo di immagazzinamento e rilascio di energia.

sembra essere stata la mancanza di coerenza nei rapporti sul guadagno di energia o sui neutroni che hanno fatto oscillare l'opinione contro qualsiasi fusione effettivamente avvenuta nella cellula F&P.


BTW - Persone che hanno continuato a farlo per alcuni anni dopo il consenso rivoltato era impegnato nella buona scienza. C'era sempre qualche piccola possibilità che il processo dipendesse da qualche fattore non misurato e non controllato. Per scoprirlo, bisognerebbe accumulare un insieme non banale di celle funzionanti e poi indagare su come fossero diverse dalle celle non funzionanti.

Ma come ha detto l'uomo "Se all'inizio tu non riuscirci, prova, riprova. Quindi esci. Non serve essere un dannato stupido. " Prima o poi devi solo arrenderti.

l'uomo ha detto "prova e prova finché non ci riesci", o almeno lo ha fatto mia madre :) Indipendentemente dai fallimenti o dalle fabbricazioni di Pons-Fleischmann o Rossi-Focardi, come fisici non possiamo rinunciare a ogni speranza di scoprire una via fisicamente praticabile per fusione, fredda o calda, che è più elegante ed efficiente dei metodi di confinamento al plasma convenzionalmente accettati.
-1 Questa risposta ignora la calorimetria. L'effetto osservato da Pons e Fleischmann era la produzione di energia al 30% al di sopra del controllo dell'acqua leggera, molti molti sigma al di sopra del loro errore calorimetrico (calibrato), sporadico nel tempo e della durata troppo lunga per essere chimica. A meno che tu non creda che l'acqua pesante sia magica, qualsiasi accumulo e rilascio di energia deve essere immagazzinato nell'elettrodo Pd ed è limitato a 1eV per atomo in un grammo di Pd, un limite che è stato superato molte volte. Queste spiegazioni sono così sciocche che è irresponsabile ripeterle.
@Ron Maimon: "Queste spiegazioni sono così stupide che è irresponsabile ripeterle." C'è l'ovvia spiegazione, che è che Pons e Fleischmann erano incompetenti. Questa spiegazione è supportata dal fatto che i loro risultati non sono riproducibili. In quanto fisico nucleare che ha eseguito la rilevazione dei neutroni, è anche dolorosamente ovvio per me che non erano competenti per la rilevazione dei neutroni.
Sembra che il palladio / deuterio possa quindi essere usato come un magnete incompetente, perché gli incompetenti Paneth / Peters (1924!) + Filimonenko (1956!) + Pons / Fleischmann (1989) non sembrano evitarlo. Anche ignorando le riproduzioni da quando, mettendomi le scarpe del 1989, non c'è possibilità che questo sia falso. Tu dovresti vergognarti di te stesso.
@BenCrowell: Capisco che il rilevamento dei neutroni sia un'arte quanto una scienza. Fleischmann e Pons erano chimici. Non riesco a immaginare che pensassero di essere bravi nel rilevamento dei neutroni, e sicuramente non sapevano in cosa stavano entrando. Avrebbero dovuto cercare aiuto, ed è stato un grosso errore per loro provare a farlo da soli. Hanno tentato di spedire il loro apparato ad Harwell nel 1989 dove si potevano effettuare misurazioni sofisticate, ma le dogane hanno impedito che accadesse prima che tutto crollasse intorno a loro. Vedere Chiudi per i dettagli.
La calorimetria di F&P non è stata impeccabile. Nel mio white paper (rif. Altrove in questo forum), ho affrontato diversi problemi con esso, e sostengo che sia stato ignorare o non rendermi conto di questi problemi che ha portato F&P a dare troppa fiducia alla loro calorimetria. E sono sicuro che inizialmente hanno cercato di rilevare l'emissione di raggi gamma, pensavano di averla trovata, ma mancava il bordo Compton, che mostrava che non avevano eseguito correttamente la spettroscopia, e hanno ritirato quella specifica affermazione.
Jerry Schirmer
2011-01-25 04:44:54 UTC
view on stackexchange narkive permalink

Ci sono alcuni motivi.

  1. Non c'è mai stata una ragione chiara per cui l'elettrificazione del palladio dovrebbe creare pressioni sufficienti per innescare una reazione di fusione. Senza un meccanismo, questa sembra essere la conclusione più ridicolmente radicale e sensazionale possibile, anche se la calorimetria dice che il palladio elettrificato crea energia netta in qualche modo. Perché non iniziare con spiegazioni più semplici che dire che è fusione?

  2. l'unica prova che qualcuno ha mai offerto è stata la calorimetria di un esperimento su piccola scala. Ma la calorimetria è una cosa complicata: devi modellare correttamente l'isolamento del sistema e devi misurare correttamente l'apporto di calore per il sistema e così via.

  3. Nessun prodotto di fusione è mai stato osservato: la fusione sarebbe plausibile se fosse in grado di dimostrare che le reazioni hanno prodotto elio-3 o litio o altro.

  4. La configurazione che descrivono in questi esperimenti sembra abbastanza semplice: basta immergere un po 'di palladio sott'acqua e far passare l'elettricità. Se questo produce davvero una reazione di fusione, perché non aumentarla e costruire un reattore? Sono sicuro che se potessero alimentare un edificio con un reattore a fusione al palladio (o anche solo una lampadina), ogni critico starebbe immediatamente zitto e si metterebbe dietro di loro. E non sembrano mai spiegare perché non lo fanno. Non crederò mai alla tua nuova forma rivoluzionaria di energia a basso costo a meno che non inizi a produrre energia a basso costo con essa (o almeno spieghi perché non puoi).

"" palladio elettrizzante "" non è corretto. Hanno elettrolizzato una soluzione acquosa utilizzando un catodo di palladiom. Il palladio (e alcuni altri metalli del gruppo del platino) producono una sorta di composto con l'idrogeno. L'idrogeno viene scisso in atomi e questi atomi si diffondono attraverso il palladio. (Questo effetto è la base per l'uso del Pd come catalizzatore di idrogenazione) Questo è lo sfondo per pensare / sperare che ci possa essere una certa "compressione" del
Modificato Modificato Modificato
@Gerog: il tuo commento è stato tagliato, ma abbastanza giusto. Ne ho letto solo a livello popolare, dove hanno fatto sembrare che ci fosse un circuito di palladio che è stato poi sommerso. Tuttavia, qualsiasi meccanismo chimico che provochi la fusione è alquanto approssimativo, considerando che le energie chimiche sono molto inferiori a quelle nucleari - mi aspetto che i legami Pd-H si rompano prima che l'idrogeno venga compresso abbastanza da fondersi. E ancora, questo non risponde alla domanda sul perché non si limitano a costruire il dannato reattore.
Ciao Jerry, questa domanda è facile: non funziona! A parte tutti i problemi di quell'esperimento, dopo un po 'di tempo dovrebbe esserci una situazione stazionaria con più energia in uscita di quella alimentata. Sono sicuro, hanno provato duramente, forse per alcune settimane ....
-1 L'esperimento è molto difficile poiché richiede 1-1 caricamento di deuterio nel Pd, e l'effetto è sporadico e sensibile alla metallurgia. La calorimetria, invece, è banale, perché puoi eseguire un controllo dell'acqua leggera e conoscere i tuoi errori. La calorimetria è una scienza affidabile vecchia di 100 anni.
In realtà, ci sono rapporti pubblicati sulla fusione fredda nel platino, che non assorbe alcun idrogeno, il che abbatte l'idea che per ottenere la fusione fredda sia necessario un elevato carico di idrogeno alla rinfusa. Allo stesso modo, è noto che Ni non carica alcuna quantità apprezzabile di idrogeno fino a molti Gigapascal di pressione applicata. Inoltre, le prime ricerche sul campo avevano apparente energia in eccesso con i catodi di Pd, sostenendo solo un carico di 0,78 H / Pd. L'H / M deve essere uguale a 1 rivendicazione originata dal lavoro di McKubre, ma come notato questo è contro esperienza precedente e correlata.
@KirkShanahan: È probabile che il materiale Pt sia negli strati superficiali, e il Ni sia _all_ errore di ricombinazione (forse anche Pt). Il motivo per cui si presta molta attenzione a McKubre è perché ottiene in modo affidabile schifezze nucleari, He, ed è attento a quantificare l'He e controllare la ricombinazione. Nella teoria che ho dato, richiede assolutamente un carico di massa elevato per osservare gli effetti nucleari, sono necessarie sezioni 1-1 omogenee per ottenere un buon d-banding senza dispersione, per quanto mi riguarda, tutte le osservazioni di calore in eccesso in Ni H o in Pt dovrebbe essere confermato con trizio e trasmutazioni prima di accettare.
@KirkShanahan: l'effetto, se esiste, è spesso pensato per essere una superficie piuttosto che un effetto di massa. È vero che Fleischmann e Pons cercarono di caricare il catodo di palladio con deuterio e avevano una serie di ragioni per questo che presumeva un effetto di massa. Ma questa potrebbe essere stata una falsa pista. Quindi è possibile che la permeabilità sia irrilevante.
-1 Ci sono molte segnalazioni di "ceneri nucleari": elio-4, trizio, bassi livelli di neutroni sopra lo sfondo, particelle alfa veloci e protoni e raggi X. È possibile che questi rapporti siano tutti errati o inadeguati. Ma dovremmo affrontare il dubbio delle relazioni piuttosto che dare un'affermazione generale, come in (3) sopra, che non sono mai stati trovati prodotti di fusione.
@Ron - Beh, almeno hai ottenuto metà del mio punto. Sì, il dibattito è stato sempre su "superficie o massa". Il lavoro di Pt e Ni mostra che è superficiale. Solo perché il Pd idruro più facilmente non si dovrebbe presumere un meccanismo diverso. E nota ancora, è stato riportato che PdHx con x <0,8 mostra gli effetti, quindi> = 1.0 NON è richiesto.
@Eric - Fin dall'inizio ci sono state affermazioni di ceneri nucleari rilevate, e dall'inizio queste affermazioni sono state contestate. Due punti generali. (1) la riproducibilità è inesistente, (2) i metodi analitici non vengono mai spiegati e alcuni risultati specifici di Clarke suggeriscono che hanno davvero, davvero bisogno di esserlo. Fino a quando i punti (1) e (2) non vengono presi in considerazione, le rivendicazioni non sono convincenti.
@KirkShanahan: Va bene che le persone non trovino convincenti le affermazioni sui prodotti di fusione. Ma le persone dovrebbero cercare di avere una conversazione, affrontando dettagli specifici, piuttosto che fare affidamento su affermazioni generali con cui il pubblico dovrebbe essere in linea (non c'erano prodotti di fusione, la riproducibilità è inesistente, i metodi analitici non sono spiegati). Tu stesso vedi la necessità di questo, poiché sei entrato in un ammirevole livello di dettaglio altrove. Harwell, MIT e Caltech hanno visto la necessità di questo.
@Eric - E il vero motivo per cui la fusione fredda dovrebbe essere considerata fasulla è che i fusioneers freddi si rifiutano di farlo. Fin dall'inizio, i risultati di He sono stati contestati come perdite, ma nessuno di loro ha riportato i livelli di fondo dell'aria di laboratorio durante il periodo sperimentale. 12 anni fa ho fatto circolare un manoscritto, successivamente pubblicato, che mostrava come i segnali di calore a livello di Watt potessero essere un errore, ma il campo non ha nemmeno ammesso che l'ho fatto, per non parlare di aver cambiato il modo in cui fanno gli estratti. L'attuale raccolto di cold fusioniers è semplicemente troppo parziale per discuterne ragionevolmente.
Per quanto riguarda il caricamento: non sono convinto che qualsiasi esperimento tranne quelli che vedono prodotti nucleari sia nucleare affidabile. Io voglio Lui e voglio il trizio e, meglio ancora, le trasmutazioni. I deuteridi di palladio caricati con .85 sono completamente caricati in superficie, il deuterio si diffonde e qualsiasi sezione completamente deuterata, anche se piccola, è potenzialmente un sito in cui è possibile la fusione. Il problema della superficie non è così significativo, ma il problema dei protoni è: non c'è modo in cui nessuno degli esperimenti sui protoni sia nucleare affidabile, non puoi superare la barriera di Coulomb. Ma questi non vedono la cenere.
@EricWalker: Non penso che vada bene non trovare convincenti le affermazioni sui prodotti di fusione: questo non è ragionevole, poiché richiede Bocris, Wolf, la gente di Bhabha, Mizuni, Iwamura e una dozzina di altre persone che hanno scoperto che il trizio si è impegnato vera e propria frode. Ci sono solo due opzioni: Bocris ha commesso una frode o avviene la fusione fredda. Non esiste una terza opzione. Questo è il motivo per cui Bocris è stato educato per violazioni dell'etica: i suoi risultati sono _impossibili_ senza fusione fredda. Non sono necessarie ulteriori conversazioni secondo me, la negazione è tutta politica a questo punto e nessuna scienza.
No Ron, nessuno ha riferito di livelli atmosferici 10X per quanto ne so. Il migliore è ~ 2x per quanto ricordo. Ma atmosp. i livelli non sono rilevanti. Gli expt. sono stati condotti in laboratori, e l'aria del laboratorio può avere Lui dentro dal ragazzo della porta accanto che sta controllando le perdite! E ancora mi chiami disonesto. Come mai? I dati che ho analizzato li ho fatti assumendo che non siano eccedenti, e l'ho affermato chiaramente. Il resto è solo il risultato diretto dell'algebra, nessuna scelta da parte mia come hai suggerito altrove. Ho appena esaminato i risultati e aggiunto una spiegazione convenzionale dettagliata. Disonesto? Non credo proprio.
In queste celle ai livelli riportati può facilmente provenire da perdite. * Chiunque * riferisca risultati simili rientra nella stessa valutazione. Mostrami come controllare in modo prevedibile il "prodotto" Lui e io potremmo credere.
@Ron: Sono decisamente d'accordo con questo punto di vista. :)
re: Transmut. - Dai un'occhiata a Rolison (Anal.Chem.63 (1991) 1697) per es. - uno studio XPS del Pd elettrolizzato in D2O e H2O. Scoprono che le impurità M alla rinfusa sono migrate in superficie in entrambi i casi. I fonditori freddi affermerebbero la trasmutazione come fonte, ma Rolison è molto più onesto. Notano anche di aver rilevato D2 che esce da Pd un mese dopo aver smesso di usarlo, il che è rilevante per le affermazioni basate sulla SIMS di spostamenti isotopici. La migliore spiegazione degli elementi estranei sulla superficie è la contaminazione. Si noti che S. Little ha mostrato che la lisciviazione dalle parti cellulari era fonte per alcuni. T e He già.
La linea di fondo è che per qualsiasi metodo tu scelga di selezionare i casi di CF * reali *, i risultati effettivi sono troppo irriproducibili per essere tutt'altro che suggestivi. Detto questo, i livelli osservati sono anche facilmente raggiungibili convenzionalmente, il che lascia (presumibilmente) due valide spiegazioni per i risultati, una convenzionale e una nucleare. Ma quella nucleare richiede nuova fisica, quella convenzionale no. La scelta "normale" in questo caso è la spiegazione convenzionale. I CFer non lo prenderanno nemmeno in considerazione. Non darei loro un centesimo solo per questo motivo.
@KirkShanahan: Stai facendo politica sciocca. Non è possibile ottenere elementi radioattivi (rilevati dalla gamma) da un catodo non radioattivo. Nessuna segregazione può produrre trizio e sì, i livelli di codeposizione di He sono stati 10 volte quelli atmosferici. La fusione non richiede nuova fisica, ho spiegato come avviene, è richiesta dalla fisica esistente, e si può prevedere che avvenga regolarmente in qualsiasi metallo sufficientemente pesante capace di deuterazione, e non c'è modo di evitarlo. Gli esperimenti sono eccellenti per i loro livelli di finanziamento e le tue pseudo-spiegazioni dei risultati sono patetiche.
Bel fallimento Ron. Chiedo uno schema dettagliato e ragionato di uno solo dei miei "errori" e ottengo quanto sopra ... P.S. Non credo alla tua teoria. P.P.S. Non credo 10X atmosferico - cita il tuo rif. per favore. Oh, e P.P.P.S. le perdite porteranno infatti ad un aumento di He nel tempo.
Potrebbe essere utile dare una rapida occhiata ad alcuni numeri. Dal libro di Storms del 2007, Tabella 6, risultati del trizio: Bockris et al 1989 2x10e5 x bkgrd; Inyengar et al (BHARC) 1989 8 x 10e15 --- è giusto 10 ordini di grandezza di differenza. Mizun_o_ e Iwamura sono più conosciuti per la trasmutazione dei metalli pesanti. vedi commento successivo.
Iwamura 2000 Pd / LiOD + D2O F, Al, Si, S, Cl, C, Ca, Ti, Cr, Mn, Fe, Co --- Iwamura 1998 Pd / LiOD + D2O Ti, Cu, Fe (cambio isotopo) - - Mizuno 1996 Pd / LiOH + D2O Pt, Pb --- Rolison 1991 Pd / LiSO4 + D2O, H2) Rh, Ag ----- davvero coerente giusto ... la mancanza di consistenza indica davvero la contaminazione dovuta a vari materiali di diversa provenienza. E ricorda che Rolison ha attribuito i loro elementi alle impurità Pd sfuse concentrate sulla superficie
@KirkShanahan: Le trasmutazioni Ag, Ru sono significative perché vengono rilevate radiologicamente da Wolf utilizzando emissioni gamma. I risultati di Bocris sono stati molte volte precedenti e vengono eseguiti in un laboratorio indipendente, che sa cosa ha fatto esattamente Inyengar. Mi affido ai dati di Iwamura, perché si concentravano su grandi trasmutazioni, non so nulla di Rolison. So solo cosa prevede la teoria, predice la frammentazione di protoni, alfa o l'espulsione di nuclei altamente stabili (gusci pieni) come appropriato per l'eccitazione 20MeV di un nucleo di Pd, e questo produce atomi di Pd ridotti di 4,8,12,16 massa unità.
@KirkShanahan: Non mi piace il modo in cui hai confrontato esperimenti accurati con un lavoro disattento, puoi screditare qualsiasi cosa confrontandoti con i risultati peggiori. Sono d'accordo sul fatto che ci siano documenti che non sono eccezionali sul campo, come in qualsiasi campo, ma bisogna cercare i dati analizzati più attentamente e affrontarli, e non rifiutarli perché qualcun altro ha fatto anche un pessimo esperimento.
Non sto contestando che i ricercatori trovino elementi "nuovi", sto contestando la loro fonte. Non è necessario assumere nuc. rxs., la migrazione della contaminazione risolve bene il problema, ed è stato dimostrato altrove (Little e Rolison per es.). Quali risultati Bockris? Il 10X che hai notato nell'altro thread? Vedi la mia risposta lì, 10X non è reale. In realtà lo stesso. Per quanto ho visto, non ci sono esperimenti accurati che finiscono per rivendicare nuc. rxs. Quelli attenti scoprono che è contaminazione.
@RonMaimon: Ho aperto una chat room per continuare questa linea di domande - Non sono sicuro che avrai ottenuto il mio ping.
user4552
2011-08-19 01:40:31 UTC
view on stackexchange narkive permalink

Pons e Fleischmann hanno originariamente riferito nel 1989 che le loro cellule chimiche avevano prodotto calore in eccesso, neutroni e trizio. La loro interpretazione era che i nuclei di deuterio si stavano fondendo per produrre 4He. I rapporti di ramificazione in questo processo sono noti: 50% n + 3He, 50% p + 3H e 10 ^ -6 4He + gamma. Se il calore in eccesso dichiarato fosse stato prodotto dalla fusione, allora gli sperimentatori sarebbero stati uccisi dai neutroni provenienti dal 50% dei decadimenti che procedevano per emissione di neutroni. I rivelatori di neutroni sono famigerati tra i fisici nucleari per essere difficili da usare e per avere la tendenza a produrre segnali spuri. Anche se tutti i conteggi di neutroni dichiarati da Pons e Fleischmann fossero stati reali, il flusso di neutroni osservato sarebbe stato di molti ordini di grandezza troppo piccolo rispetto alla quantità di calore in eccesso dichiarata. Successive misurazioni effettuate da fisici nucleari utilizzando tecniche di rilevamento dei neutroni all'avanguardia hanno dimostrato che nessun neutrone al di sopra del livello di fondo è prodotto da cellule chimiche del tipo utilizzato da Pons e Fleischmann. [Gai 1989] Non sono state inoltre fornite prove plausibili per la produzione. di raggi 3He, 3H o gamma, che sarebbero stati tutti copiosamente prodotti in reazioni di fusione dd che producevano quantità misurabili di energia attraverso reazioni nucleari.

A partire dal 2010, il consenso tra gli scienziati è che la fusione fredda fosse un esempio di scienza patologica. Tuttavia, alcuni veri credenti continuano a fare esperimenti e ad affermare risultati positivi. Una recensione del 2010 di un eminente credente [Storms 2010] afferma che "Molte persone ritengono che la correlazione tra calore ed elio sia la prova più forte della fusione fredda". Il problema è che il calore è comunque prodotto da reazioni chimiche e i livelli di elio dichiarati non sono abbastanza alti da mostrare un eccesso convincente rispetto allo sfondo. Se queste affermazioni fossero corrette, richiederebbero anche una fondamentale riscrittura delle leggi della fisica. Richiederebbero che il rapporto di ramificazione nella fusione d-d venga drasticamente alterato dall'ambiente chimico, ma questo è impossibile perché in una reazione nucleare, gli elettroni sono semplici spettatori. Per conservare energia e quantità di moto, la fusione d-d richiede anche l'emissione di due particelle nello stato finale. Per aggirare l'inesistenza della seconda particella, gli entusiasti della fusione fredda suppongono che l'energia della reazione venga trasmessa al reticolo elettronico. Non esiste alcun meccanismo noto mediante il quale tale trasmissione potrebbe avvenire. Man mano che i vincoli sperimentali e teorici si sono irrigiditi, i credenti hanno risposto inventando una pseudoscienza più folle, includendo la produzione di nuclei con numero atomico 126 e la trasmutazione degli elementi da parte di piante e batteri. [Storms 2010]

In sintesi, le affermazioni sulla fusione fredda non possono essere corrette a meno che non ribaltino le conoscenze saldamente stabilite della fisica nucleare. Questa sarebbe un'affermazione straordinaria e richiederebbe prove straordinarie. Dopo due decenni, non è emersa alcuna prova così straordinaria.

Gai et al., "Limiti superiori sull'emissione di neutroni e raggi gamma dalla fusione fredda", Nature 340 (1989) 29-34.

Storms, "Status of cold fusion (2010)", Naturwissenschaften (online) 97 (10): 861–881

La tua risposta è piuttosto equilibrata, ma ti consiglio di guardare gli esperimenti SPAWAR e gli esperimenti di Arata. SPAWAR mostra in modo conclusivo che ci sono particelle cariche energetiche, mentre Arata mostra che il calore e l'elio sono correlati. I livelli di elio in diversi esperimenti di tipo Pons / Fleischmann sono ben al di sopra del livello di fondo e la produzione di trizio è completamente inspiegabile. Per quanto riguarda il trasferimento di energia nucleare al reticolo, questo è un mistero che potrebbe essere risolto in linea di principio da un'interazione deuterone-deuterone-nucleo a tre corpi, quindi non esclude il processo.
Il documento Gai che citi: http://www.nature.com/nature/journal/v340/n6228/abs/340029a0.html non fa calorimetria. Come avrebbero potuto sapere quando stava accadendo la reazione, se non del tutto? Forse hanno funzionato male. Forse hanno funzionato a un carico troppo basso. È un giornale spazzatura e non avrebbe mai dovuto essere pubblicato.
@Ron Maimon: Ovviamente siamo completamente in disaccordo su cosa sia la scienza dei rifiuti e cosa non lo sia. "Forse hanno funzionato male." Ovviamente hanno funzionato. La fusione fredda non esiste, quindi tutte le celle di fusione fredda sono stupide.
Se non sono in grado di riprodurre l'effetto, che valore aggiunge alla misurazione dei conteggi di neutroni di fondo? Volevano solo aggiungere le loro voci alla lista "anch'io" delle riproduzioni fallite. Ho mostrato a mio padre, dai modi gentili, carte come queste e gli ho detto: "Sai, gente del genere dovrebbe essere licenziata". Ha detto che "persone del genere dovrebbero essere fucilate".
Per chi ha problemi di umorismo, il vero aneddoto sopra è uno scherzo, non una minaccia.
anna v
2011-02-01 13:53:12 UTC
view on stackexchange narkive permalink

Questa nuova "fusione fredda" riportata in quello che è veramente un blog è un'impresa commerciale a tutti gli effetti. Le loro pretese sono così grandi che i loro costrutti avranno successo o mangeranno il loro cappello. Non dobbiamo aspettare molto.

Se avranno successo, la teoria verrà trovata.

Una nota sui cristalli (usano cristalli di Ni) e le grandi energie: lungo l'asse dei cristalli i fasci di muoni ad alta energia passano intatti, senza interagire con la barriera coulombiana. Difficile trovare riferimenti, so che Tom Ypsilantis ci stava lavorando negli anni '80. Ecco una proposta per un collisore di muoni che utilizza il concetto. Quindi, se l'annuncio non è acqua di serpente, emergerà un'appropriata teoria dello stato solido / nucleare.

Sto modificando questo per includere un recente video della NASA che potrebbe dire che le affermazioni di Rossi potrebbero non essere del tutto false. Notare il Ni28 nella tabella dei possibili elementi da utilizzare per la fusione nucleare a bassa energia (LENR).

BTW Rossi e altri stanno ora progettando piccoli riscaldatori! dimensione del laptop per il mercato interno !! Se si tratta di una truffa, sarà la truffa a porre fine a tutte le truffe!.

Uno dei professori che avevo tempo fa era Joseph Weber. Era convinto che i cristalli potessero catturare un'alta sezione trasversale di neutrini. Ricordo che ha convinto la Marina a finanziarlo. Ha ottenuto risultati positivi che nessuno ha mai ripetuto. A quel tempo, i dottorandi cercavano sostanzialmente di evitarlo. Ma era un bravo ragazzo. Google "joe weber" + cristallo + neutrino
questo [brevetto della NASA] (http://appft1.uspto.gov/netacgi/nph-Parser?Sect1=PTO1&Sect2=HITOFF&d=PG01&p=1&u=%2Fnetahtml%2FPTO%2Fsrchnum.html&r=1&f=G&l=50&25% 22.PGNR. & OS = DN / 20110255645 & RS = DN / 20110255645) l'esplorazione dei "polaritoni plasmonici" e degli "elettroni pesanti" sembra essere correlata a quel video della NASA.
questo [articolo del capo scienziato della NASA] (http://futureinnovation.larc.nasa.gov/view/articles/futurism/bushnell/low-energy-nuclear-reactions.html) "Sta succedendo qualcosa di reale ... NASA LaRC è iniziato Studi di progettazione LENR guidati da ... "(trovato tramite il blog 22passi)
AbstractDissonance
2011-01-25 08:45:28 UTC
view on stackexchange narkive permalink

Molto semplice! Non possono mai ripetere i risultati in modo scientifico per dimostrare agli altri che funziona. Qual è il punto della scienza se ignoriamo semplicemente il metodo scientifico? Se hanno davvero inventato qualcosa, non avrebbero bisogno di essere riservati e non mostrare esattamente quello che hanno fatto. Se in qualche modo l'hanno ottenuto per caso, allora non è scienza. Una volta che sono in grado di riprodurre i risultati e mostrarli agli altri e gli altri possono riprodurli, allora diventa utile e diventa scienza. (anche se la fusione fredda è possibile è inutile se non riusciamo mai a riprodurla)

Il motivo per cui la fusione fredda è considerata fasulla è principalmente perché molte persone hanno provato e tutti hanno fallito ... e quelli che hanno affermato di avere successo non hanno mai dimostrato di averlo fatto.

Nessuno sa se la fusione fredda sia possibile, ma visti tutti gli schemi e gli svantaggi che sono accaduti in passato è più facile essere scettici che no. Se qualcuno scopre un modo, allora sarà MOLTO facile da dimostrare e se sono veri scienziati non avranno problemi a farlo.

Penso che ci siano altri motivi migliori di questo. Ad esempio, i primi laser avevano difficoltà con la ripetibilità, fino a quando altri non hanno appreso le tecniche sottili relative all'impedenza del cavo di alimentazione, ecc. In generale, qualsiasi cosa nuova è sperimentalmente difficile e se non sai come funziona avrai difficoltà a ripeterlo perché non sai esattamente cosa stai ripetendo. (Cioè il colore della vernice è importante? Potrebbe se c'è una sostanza chimica nella vernice che è fondamentale.)
Um, no, sei totalmente fuori luogo. Non sto parlando della ripetibilità delle misurazioni ma di qualcosa che le persone esperte chiamano metodo scientifico ... imparalo e rileggi quello che ho detto.
Il metodo scientifico (sm) non aveva nulla a che fare con le passate rivoluzioni scientifiche di Newton, Einstein, .. (non sto dicendo che sm sia irrilevante per i piccoli passi incrementali). L'intuizione e il duro lavoro contro l'establishment sono la norma.
Kirk Shanahan
2012-11-06 23:38:31 UTC
view on stackexchange narkive permalink

Perché la fusione fredda è stata considerata fasulla? Perché non era facilmente riproducibile quando inizialmente annunciato, perché il meccanismo suggerito originale era incoerente con la fisica nota all'epoca e perché le prove presentate all'epoca che pretendevano di dimostrare che si trattava di fusione nucleare (in particolare fusione DD) erano difettose.

Forse la domanda migliore è: dovrebbe essere considerato fasullo oggi? No, non completamente. Sono disponibili molti casi di risultati simili, ma l'effetto non è ancora sotto controllo. La ragione migliore è che i ricercatori stanno tentando di controllare le cose sbagliate quando fanno i loro esperimenti, e questo finisce con molti risultati diversi a seconda dei valori che i fattori di controllo reali hanno assunto durante il sperimentare.

Ci sono spiegazioni convenzionali per quei risultati semi-riprodotti che suggeriscono altre cose da controllare, ma che i ricercatori della fusione fredda non riescono a impiegare perché rifiutano di considerare soluzioni non nucleari. Per maggiori dettagli, vedere l'URL di seguito, leggerlo e i relativi riferimenti:

https://docs.google.com/open?id=0B3d7yWtb1doPc3otVGFUNDZKUDQ

(rifatto in http://www.networkworld.com/columnists/2012/102612-backspin.html?page=1)

-1: la mancanza di una piena conoscenza dei fattori di controllo non è sufficiente per negare un effetto nucleare. Tutto ciò di cui hai bisogno per un effetto nucleare è _una_ misurazione affidabile di una trasmutazione nucleare di qualsiasi tipo. Le misurazioni del trizio sono sufficienti per dimostrarlo, non è necessario altro. Si possono fare buone domande sui fattori di controllo solo _dopo_ che si riconosce che ci sono fenomeni nucleari in questi sistemi. Non bisogna capire tutto per capire qualcosa (anche se ormai credo di aver capito tutto). Si può essere sicuri che non si può produrre trizio con la chimica non nucleare.
No, devi essere certo, oltre ogni ragionevole dubbio, che i materiali che stai misurando non siano arrivati ​​in modo convenzionale. Sfortunatamente, questo non è mai stato sufficientemente dimostrato. I risultati estremamente variabili ottenuti dai CFers in realtà implicano che non stanno controllando i veri fattori di controllo, e dopo 25 anni degli stessi cattivi risultati, è tempo di provare ad abbaiare su un albero diverso. Dacci un'equazione che dica: "Se aggiungi x quantità di A e riscaldi a y gradi per z ore, otterrai B a tale livello +/- 10%". per rispondere finalmente alla domanda.
Questo non è un modo ragionevole per valutare un fenomeno sporadico e mal compreso. Quello che vuoi è una cosa statistica --- Se ordino 100 diversi lotti di Pd, li carichi con deuterio, rileverai il trizio a livelli molto al di sopra del fondo trascurabile nel 5% dei campioni. Questo dipende da fattori che le persone non capiscono, secondo me la cosa più importante è un'adeguata contaminazione radiologica del Pd, che dà il seme per l'inizio della reazione. Se leghi Pd con un buon emettitore alfa, potresti essere in grado di creare un campione riproducibile al 100% (sebbene potrebbe anche esplodere).
Inoltre, ho visto un breve commento sull'irradiazione a raggi X a 115 KeV di Pd protonato da qualche parte da parte tua (non riesco a trovarlo di nuovo), per rispondere a questo: la banda che mi aspetto sia intorno a 20-40 KeV (non posso dire esattamente dove, perché sono modalità miste di deuterone e guscio interno, e dipende dalla massa del deuterone, ma in nessun modo è vicino a 115 KeV). Ciò significa che l'irradiazione dei raggi X a frequenze 3-4 volte più alte è inutile, sono necessari esperimenti a raggi X vicino al guscio K del Pd, per trovare il picco K che si allarga e si sposta in una banda con un bordo di banda da qualche parte vicino al vecchio K -conchiglia.
Dati riproducibili esigenti sono la pietra angolare della scienza. I dati affidabili sono dati riproducibili. Non è possibile ottenere un risultato unico e affidabile a causa di possibili interferenze, il che significa che non puoi provare che provenga da ciò che stai sostenendo che provenga. Devi dimostrare che i fattori proposti sono importanti con i dati. Con abbastanza dati si può iniziare a parlare di affidabilità. I numeri estremamente variabili indicano che non hai controllato alcuni fattori importanti. Per identificare i fattori, i candidati ragionevoli NON devono essere arbitrariamente ignorati. I CFer bloccano i fattori convenzionali. I risultati variabili indicano che era male.
BTW, un ulteriore commento su questa cosa della teoria del Maimon K-shell. Il più delle volte non si osserva alcun effetto CF, si devono creare circostanze speciali e non ho mai sentito parlare di CFer che espongono apparecchi a radiazioni keV. Allora come si fa _ normalmente_ a ottenere CF da questa cosa dell'elettrone della shell K?
Stai confondendo fotoni e particelle cariche. I fotoni devono essere sintonizzati, le particelle cariche producono fori keV senza messa a punto, questa è la ionizzazione della formula di Bethe. Il limite di una banda è che è necessario concentrare i fori in modo sufficiente in una regione per consentire la fusione, ciò potrebbe richiedere un forte campo elettrico superficiale, concentrando i deuteroni per consentire la fusione d-d (la cui velocità è quadratica nella densità d). È inoltre necessario seminare la reazione, quindi è necessaria una quantità di traccia di radioattività delle particelle cariche sul Pd. Questo alla fine è inevitabile, la sporcizia si accumula sul catodo.
Ci sono diverse possibili ragioni per "condizioni speciali", potresti aver bisogno di tracce di emettitori alfa sul catodo, che potrebbero variare da campione a campione o dal tipo di soluzione, potresti avere raggi cosmici diversi o radioattività di fondo, che potrebbe essere l'innesco , oppure potrebbe essere dovuto ai campi elettrici superficiali richiesti per concentrare il foro del deuteron-k-shell in una densità sufficiente per innescare la reazione. Chissà. Poiché persone come te hanno impedito intenzionalmente di ricercare l'argomento, c'è molto poco da dire in modo definitivo. Ma ora sono sicuro che il mio meccanismo sia corretto.
E perché non si dovrebbe considerare un fascio di elettroni un fascio di particelle cariche? Sparare un fascio di elettroni di 2-3 keV su un metallo produce elettroni Auger (e talvolta anche gamma) in prossimità di 1-1000 eV, chiaramente non una condizione di risonanza, dai buchi dei gusci K e L e il loro riempimento con elettroni di valenza; una tecnica analitica superficiale molto ben definita. Penso che i tuoi criteri e la tua teoria siano fuori base, ma ammetto di non essere qualificato per diagnosticare il tuo problema. dovrai chiamare qualcun altro.
Sì --- un raggio di elettroni funzionerebbe se fosse a 100KeV-1MeV, ma preferibilmente un LINAC commerciale a 20 MeV, questo dovrebbe essere eccellente per stimolare la fusione fredda. Il processo Auger è quello che sto sfruttando, tranne per il fatto che si ottengono deuteroni Auger anche in un metallo deuterato, non solo elettroni Auger, e che i deuteroni Auger sono delocalizzati e si fondono, e la fusione semina più deuteroni Auger attraverso il suo guscio K buchi. Questo è il punto principale. Sono contento che tu stia lottando con esso, ma ti incoraggio a continuare a leggere e continuare a criticare, come vedrai non ci sono problemi con la mia idea.
Lawrence B. Crowell
2011-01-25 06:20:27 UTC
view on stackexchange narkive permalink

Se ci fosse stato un processo di fusione ci sarebbe stata anche la produzione di neutroni. C'erano alcune affermazioni sui neutroni, ma non sono state verificate. C'è un altro problema con i neutroni, se ci fosse la fusione Pons e Fleischmann sarebbero stati irradiati dai neutroni. Potrebbero infatti essersi ammalati o morti. C'era qualcosa di sbagliato in tutta la faccenda dall'inizio. Se intendevano ottenere la fusione fredda, perché non si sono difesi? Se non intendevano ottenere la fusione fredda, ma in seguito sospettavano di averla, perché non si sono quindi schermati contro i neutroni? Hanno giocato con questo set per un po 'di tempo, e se pensavano davvero di ottenere la fusione, perché non hanno installato l'apparato dietro i mattoni di piombo? O erano stupidi o truffavano.

L'idea che il potenziale reticolo interionico possa costringere i nuclei a riunirsi è ridicola. C'è una prospettiva per una fisica di bosonizzazione o condensato. Qui la D o la T entrano nello stesso stato quantistico e potrebbero passare a H ^ 4. Questo però non è plausibile.

"" perché non hanno posizionato l'apparecchio dietro i mattoni di piombo? O erano stupidi o stavano truffando. "" Anche il piombo per proteggere i neutroni non è poco stupido.
Non so quale sia il potere frenante del piombo sui neutroni. Sembra che tu stia insinuando che il piombo non funzionerà. Tuttavia, un moderatore, forse grafite, o qualcosa per fermare i neutroni sarebbe prudente se stai facendo qualcosa che li generi.
I neutroni perdono energia per contatto diffondendosi con i nuclei e perdono una frazione maggiore a ogni dispersione se il nucleo è leggero. Quindi spesso si usa acqua o plastica. L'aggiunta di boro, cloro, gadolinio o qualcos'altro con una sezione trasversale di cattura termica elevata può aiutare, ma genererà gamma che devono essere schermati con uno strato Z elevato. Fondamentalmente i neutroni sono un dolore e il piano migliore è mantenere le distanze.
Lo scudo neutronico classico è la cera di paraffina, oggi PE o PP potrebbero essere usati con lo stesso effetto. I protoni nell'idrogeno della paraffina fanno lo scattering. L'acqua ha bisogno di un contenitore, non parrafin.
adechiaro
2011-02-01 10:02:37 UTC
view on stackexchange narkive permalink

In realtà, per quanto ne so, gli scienziati sono stati in grado di riprodurre alcuni esperimenti con successo, ma non hanno ancora compreso appieno il processo. È in corso una ricerca per elaborare una teoria LENR funzionante (ciò che è noto come nella comunità dei fisici) per modellare accuratamente ciò che sta accadendo (generazione di calore in eccesso inspiegabile). Come alcuni hanno detto di sì, è un processo molto complesso ed è ancora possibile che l'energia sia liberata da altri mezzi (non nucleari) ma difficilmente direi che è stato completamente provato o in altro modo. Un buon sito web per saperne di più è http://lenr-canr.org/

Citando il tuo buon sito web: _Non sono stati segnalati tentativi di replica da parte di altri gruppi_
No, la fusione fredda non è mai stata riproducibile in modo affidabile. Ogni volta che qualcuno competente cerca di riprodurlo, non succede. Vedere, ad esempio, Gai et al., "Limiti superiori sull'emissione di neutroni e raggi gamma dalla fusione fredda", Nature 340 (1989) 29-34.
@BenCrowell: Gai non è competente. Le persone competenti sono quelle che lo hanno riprodotto.
jimjim
2011-01-25 04:02:47 UTC
view on stackexchange narkive permalink

Aggiornamento: credo che sia lo stesso motivo per cui la teoria del multi universo è considerata falsa da alcuni: "speculazione fantastica, scollegata dalla realtà a cui possiamo accedere empiricamente"

http: // www.scientificamerican.com/blog/post.cfm?id=is-speculation-in-multiverses-as-im-2011-01-28

Vecchio contenuto qui sotto: volevo invialo come commento alla tua domanda, ma non ho abbastanza rappresentante per farlo.

Nel libro "Forbidden Science", viene discussa la fusione fredda e viene menzionato l'atteggiamento della comunità scientifica. Il libro "I problemi con la fisica" offre anche uno sguardo alla comunità dei fisici.

Forse invece di chiedersi perché è considerato fasullo, dovrebbe cambiarlo in considerato fasullo da alcuni. Non sono stato in grado di trovare una verifica alla storia che prima che gli aerei fossero inventati c'erano prove "matematiche" che fluttuavano intorno al motivo per cui era impossibile che le macchine di metallo volassero. (qualcuno ha riferimenti per questo?)

Questo è esattamente il punto: la fisica moderna ti dice che la fusione fredda è impossibile. Affermare il contrario significa o respingere la teoria supportata da numerosi fatti, o inventare nuovi tipi di interazioni che sono state miracolosamente perse da tutti i fisici con i loro acceleratori di particelle e tuttavia eccola qui, che accade nel tuo bicchiere d'acqua.
Non confondere matematica e fisica. In fisica la matematica è uno strumento, potenzialmente uno strumento in una configurazione per produrre prove. Tuttavia, la matematica non è mai una fine. L'unica cosa che conta in fisica è l'osservazione empirica, tipicamente attraverso la ripetizione di esperimenti. Quindi, la fisica non deve produrre alcuna costruzione matematica come prova del perché qualcosa è impossibile, spetta a coloro che affermano che qualcosa _è_ possibile produrre prove empiriche. Questa linea di attività potrebbe non essere conveniente per alcuni, il che non ha conseguenze per la fisica.
@gigacyan: Caro gigacyan, a Sydney c'è un edificio marittimo, ogni volta che passo davanti all'ingresso del palazzo vedo un enorme nodo intrecciato da fili d'acciaio. Nel segno accanto si dice che la persona che l'ha fatto lo ha fatto con la consapevolezza di ingegneri che sapevano chiaramente per certo che il nodo è impossibile. Eppure è così, nonostante la convinzione di molti esperti del settore, è stato fatto. La scienza non è regola delle folle, la vittoria spetta a chi non accetta ciò che NON si può fare, ma cerca di capire come farlo.
@Arjang: In generale, sono d'accordo con il tuo punto ed è successo che la maggior parte degli scienziati si fosse sbagliata su qualcosa. Ma i principali progressi della fisica nella prima metà del XX secolo l'hanno spinta a uno stato in cui i fisici possono capire tutto ciò su cui possono mettere le mani. Fanno grandi energie con LHC solo per trovare alcune nuove cose che potrebbero studiare. Non c'è posto per una nuova interazione fisica che avvenga in condizioni normali che in qualche modo passerebbero inosservate da migliaia di fisici per 100 anni.
@gigacyan: Da quello che è noto oggi, sì, è impossibile. Ma provando si imparerebbero cose nuove e si farebbero progressi. Forse non nella stessa fusione fredda, ma in altre aree. In matematica, risolvere i problemi difficili da soli non aveva molta importanza alla fine, ma nel modo in cui farlo sono stati fatti molti più strumenti / scoperte. Ad esempio, guarda l'Ultimo Teorema di Fermat, se fosse vero o no non era importante quanto i campi di studio che erano stati inventati dalle persone ci stavano lavorando. Non si tratta solo del risultato finale, ma del viaggio per arrivarci.
@Arjang: se ti piacciono gli esempi storici, prendi le macchine a moto perpetuo, i tuoi argomenti possono essere applicati anche a quelli. Diresti che non possiamo escludere la possibilità di costruire una macchina del genere perché potrebbero esserci fenomeni fisici di cui non siamo a conoscenza che lo renderebbero possibile?
Immagino che ad un certo punto debba essere tracciata una linea. Ma la domanda è chi dovrebbe tracciare la linea, noi che non li finanziamo o la comunità scientifica che è diventata il nuovo dogma. Guardando quanti soldi sono stati sprecati in guerre, il finanziamento totale della ricerca (sia scientifica che folle) non è niente. Se qualcuno decide di seguire le macchine a moto perpetuo, lo indicherei a ciò che è noto e alla storia dei tentativi e auguro loro tutto il meglio. Una macchina a moto perpetuo che utilizza l'energia del punto zero potrebbe essere una nuova svolta? http://en.wikipedia.org/wiki/Zero-point_energy#Claims_in_pseudoscience
@Arjang se, in futuro, la fusione fredda e / o le macchine mobili perpetue si saranno dimostrate possibili attraverso ripetuti esperimenti, saremo tutti corretti. Fino a quel momento non c'è motivo di ritenere che sia possibile.
@Steven Devijver: è tutto quello che stanno facendo? Ripetendo lo stesso esperimento più e più volte? Questa è follia! Pensavo che per ricerca intendessero variare, modificare, realizzare cose nuove. Ovviamente ripetere lo stesso esperimento e aspettarsi un risultato diverso è folle, ma non dovrebbe essere chiamato ricerca.
C'è una grande differenza tra fusione fredda e multiversi: la fusione fredda, se esiste, dovrebbe essere dimostrabile sperimentalmente (in effetti, Pons e Fleischmann hanno affermato di aver fatto esattamente questo), ma violerebbe le teorie attuali. D'altra parte, universi multipli non sono osservabili in linea di principio, ma non violano alcuna legge fisica nota e possono essere dimostrati falsi solo dimostrando falsa una delle teorie fisiche che dovrebbero spiegare. Quindi la questione della fusione fredda è fisica, mentre la questione dei multiversi è fondamentalmente filosofica.
Le macchine a moto perpetuo @StevenDevijver: (che funzionano) violano il CoE, per definizione. Ciò che rientra nel termine "fusione fredda" è principalmente un insieme di affermazioni empiriche che cercano spiegazioni. Le spiegazioni non banali che vengono subito in mente generalmente violano le leggi della fisica conosciute. Ma le stesse affermazioni empiriche non sono le stesse delle spiegazioni che forniamo. Ciò pone le macchine per fusione fredda e moto perpetuo in categorie molto diverse.
Helder Velez
2011-02-25 07:11:29 UTC
view on stackexchange narkive permalink

Vogliono essere ricchi e cercano di ottenere finanziamenti. ;) Due modi:

  1. Hanno "un dispositivo segreto" che funziona -> finanziamenti, brevetti, RICH, RICH, RICH ... (Non credo che abbiano questo segreto)
  2. hanno "una bugia segreta" - finanziamenti, RICCHI (era un'operazione a rischio, ecc., dettagli contrattuali, ... reputazione rovinata ... ma RICCHI)

Non darò un giudizio diretto su fasullo o non fasullo. Aspetterò e vedrò. Qualcuno potrebbe fare esperienze inaspettate come il recente "anti-laser" che "distrugge" l'energia. Per giustificare la mia posizione "operativa" posso raccontare una storia di arroganza accademica: la trasmissione televisiva nazionale è iniziata qui nel 1955, in 1957 lo Sputnic russo fu mandato in orbita e, opportunamente, un accademico apparve in TV e disse: "È impossibile, i russi sono bugiardi".

* aggiunto: * trovato:
Come trasmutare gli elementi con la luce laser

Isotopo fotonucleare coerente trasmutazione (CPIT) produce esclusivamente isotopi radioattivi (RI) da reazioni fotonucleari coerenti (γ, n) e (γ, 2n) tramite risonanze giganti E1.

EDL - Proton-21 (Adamenko)

L'obiettivo principale della ricerca di EDL si basa su un processo di nuova concezione e auto-sostenibile che porta, attraverso una stimolazione controllata, al collasso della materia condensata. In questo stato collassato così creato, l'effetto della barriera di Coulomb diventa insignificante e si verifica e si può osservare una rapida trasmutazione di elementi e isotopi.

aggiornamento arxiv 2013/05 - esperimento 1 - teoria 0
SEVEN ricercatori delle università italiane e svedesi, in particolare Hanno Essén, hanno recentemente riportato un
Indicazione di produzione anomala di energia termica in un dispositivo reattore :

Un'indagine sperimentale su una possibile produzione anomala di calore in un tipo speciale di reattore ... Una produzione anomala di calore è stata indicato in entrambi gli esperimenti. .., il risultato è ancora un ordine di grandezza maggiore rispetto alle fonti di energia convenzionali .

Apparentemente la teoria è nei guai.

Ho aggiunto un aggiornamento 2013/05 da 7 investigatori
il collegamento arxiv è stato ottenuto tramite [wavewatching] (http://wavewatching.net/fringe/the-hot-cat-report/), lo stesso sito in cui ho scoperto che il computer quantistico D-Wave funziona contro la teoria QM del consenso.
ganzewoort
2011-09-22 07:12:06 UTC
view on stackexchange narkive permalink

Questa è la pubblicazione in cui le radici degli studi di Rossi et al. sono: Noninski V. C., Fusion Technology, 21, 163-167 (1992). Apparentemente l'importante squilibrio di potere trovato in questo articolo (pubblicato in una rivista legittima sottoposta a revisione paritaria) deve essere ricercato in alcuni aspetti della fisica classica fino ad ora sconosciuti.



Questa domanda e risposta è stata tradotta automaticamente dalla lingua inglese. Il contenuto originale è disponibile su stackexchange, che ringraziamo per la licenza cc by-sa 2.0 con cui è distribuito.
Loading...